![]() |
|
|
FlashChat | Actuarial Discussion | Preliminary Exams | CAS/SOA Exams | Cyberchat | Around the World | Suggestions |
Browse Open Actuarial Jobs |
Investment / Financial Markets Old Exam MFE Forum |
![]() |
|
Thread Tools | Search this Thread | Display Modes |
#1
|
|||
|
|||
![]() Is it me or is SOA IFM sample question 15 erroneous?
It says the table is giving us the expected return and volatility for stocks X, Y and the market. Then the table column heading says we are getting the required return for stocks X, Y and the market. The problem then asks us to find the required return for stock Y, assuming CAPM holds. Isn't the required return = E[X] - risk_free_rate? In that case, the answer should be 2.52 and I don't need to find the risk free rate at all. |
#3
|
||||
|
||||
![]() Assuming the CAPM holds, the expected return of a security is equal to the required return of a security. Thus, assuming the market is in the CAPM equilibrium, these two terms are often used interchangeably.
The risk premium of X (not the required return of X) is the expected return of X minus the risk-free rate. |
![]() |
Thread Tools | Search this Thread |
Display Modes | |
|
|